16 votos

Prueba de que los estados de energía de un oscilador armónico dado por el operador escalera incluyen todos los estados

En mecánica cuántica, mientras estudiaba el oscilador armónico, aprendí sobre los operadores de escalera. Y me di cuenta de que si eres capaz de encontrar o determinar cualquier estado energético del oscilador armónico cuántico, entonces, utilizando los operadores de escalera, puedes determinar también los demás estados energéticos. Sin embargo, en ninguno de los textos que leí encontré el siguiente hecho:

Los estados energéticos determinados por el procedimiento anterior, es decir, utilizando el operador escalera son los únicos estados energéticos posibles del oscilador armónico. No existe ningún estado energético que no esté dado por el operador escalera.

Así que esta es mi pregunta:

¿Son los estados de energía determinados por el operador escalera en el caso de un oscilador armónico, los únicos estados energéticos posibles? ¿Es posible algún otro estado energético? ¿Y cuál es la prueba?

Es bien sabido que los estados mencionados son los únicos estados energéticos posibles, pero quiero una prueba rigurosa de que ningún otro estado es posible. Sin embargo el procedimiento analítico mencionado en diferentes libros muestra una solución aproximada de la ecuación de Schrodinger que puede considerarse rigurosa en el sentido de que resuelve la ecuación para derivar las soluciones aunque considera ciertas aproximaciones.

He buscado esta respuesta en libros relacionados con la mecánica cuántica escritos por DJ Griffiths, Gasiorowicz, Dirac y en recursos online como OCW, cursos de la Universidad de Columbia, caltech.edu, pero no he podido encontrar una respuesta adecuada.

1 votos

No sé a qué te refieres cuando dices que el procedimiento analítico es una solución aproximada. ¿Te refieres al análisis asintótico, donde escribes tu función de onda como $\psi(x)\sim e^{-ax^2}$ en general $x$ y luego definir $\psi(x)=f(x)e^{-ax^2}$ ? Esto no es una aproximación, es sólo una forma de redefinir tu ecuación en términos de una función más agradable $f(x)$ .

0 votos

@BobKnighton sí estoy hablando del análisis asintótico y por mucho que pueda aprender de griffiths, parece un poco una aproximación. No es exactamente lo que se llama matemáticamente riguroso.

5 votos

No es una aproximación. La solución dada es exacta, y la forma de llegar a ella es rigurosa. La parte que parece una aproximación (el análisis asintótico) es simplemente una forma de motivar los siguientes resultados. Podríamos hacer lo mismo sin el análisis diciendo: "Definir una nueva función $f(x)=\psi(x)/e^{-ax^2}$ y escribir la ecuación de Schrodinger en términos de $f$ . Entonces podemos demostrar que $f$ debe ser un polinomio terminal si $\psi$ debe ser acotado". Es perfectamente riguroso, y equivalente (aunque un poco menos elegante) al método del operador.

14voto

Anzkji Puntos 11

Esta es una pregunta fantástica. Empecemos.

Asumiré que ya hemos definido los operadores de escalera $a$ y $a^{\dagger}$ y han definido un "estado básico" $|0\rangle$ (aún no hemos demostrado que sea el estado fundamental) tal que $a|0\rangle=0$ . También supondremos que ya sabemos que el hamiltoniano del oscilador armónico puede escribirse en la forma

$$H=\hbar\omega\left(a^{\dagger}a+\frac{1}{2}\right).$$

(Obsérvese que el estado básico $|0\rangle$ es trivialmente un estado propio con $E_0=\hbar\omega/2$ .) Por último, supondré que ya hemos mostrado las relaciones de conmutación de los operadores de escalera. A saber,

$$[a,a^{\dagger}]=1.$$

Con esto, tenemos suficiente para una prueba.

Podemos definir un estado $|n\rangle$ (olvidemos la normalización por ahora) como

$$|n\rangle=(a^{\dagger})^n|0\rangle,$$

donde $n$ es un número entero no negativo. El estado $|n\rangle$ es un estado propio del Hamiltoniano con energía $E_n=\hbar\omega(n+1/2)$ . Queremos demostrar que el conjunto $\{|n\rangle\}_{n\in\mathbb{Z}^+}$ son todos los posibles estados propios normalizables del hamiltoniano.

Recordemos que en la representación de la posición, si tenemos un potencial $V(x)$ entonces no podemos tener un estado propio normalizable $|\psi\rangle$ cuya energía satisface $E_{\psi}\leq\min V(x)$ . Es decir, no podemos tener una energía menor que la energía potencial mínima del sistema (es decir, la energía cinética debe ser positiva).

Ahora, terminamos con una prueba por contradicción. Consideremos un estado propio $|\psi\rangle$ cuya energía viene dada por $E_{\psi}=\hbar\omega(n+1/2+\epsilon)$ con $\epsilon\in(0,1)$ . Dicho estado describiría esencialmente cualquiera de los "otros" estados que $H$ podría permitir. Ahora, considera el estado $|\psi^{(1)}\rangle=a|\psi\rangle$ . Por el álgebra de los conmutadores, no es difícil demostrar que $|\psi^{(1)}\rangle$ tiene energía

$$E_{\psi^{(1)}}=\hbar\omega\left((n-1)+\frac{1}{2}+\epsilon\right).$$

Ahora, podemos inducir y definir un estado $|\psi^{(m)}\rangle\equiv(a^m)|\psi\rangle$ . Claramente, su energía viene dada por

$$E_{\psi^{(m)}}=\hbar\omega\left((n-m)+\frac{1}{2}+\epsilon\right).$$

Así, a menos que este proceso termine en algún momento (es decir, $a|\psi^{(m)}\rangle=0$ para algunos $m$ ), podemos alcanzar una energía arbitrariamente baja. Sin embargo, este proceso no podría terminar nunca, ya que el estado básico $|0\rangle$ es única (está definida en términos de un operador de posición y un único operador de derivada, por lo que $a|0\rangle=0$ simplemente define una ecuación diferencial de primer orden en el espacio de posición) y tiene energía $\hbar\omega/2$ Esto no se puede lograr para cualquier $\epsilon$ en el rango dado. Por lo tanto, ningún estado de este tipo $|\psi\rangle$ puede ocurrir. Del mismo modo, no podemos tener un estado con energía $E_{\psi}\in(0,\hbar\omega/2)$ por la misma lógica.

Así, hemos demostrado (muy rigurosamente) que la única normalizable de $H$ son los que tienen energía $\hbar\omega(n+1/2)$ que se obtienen de forma única a partir de la acción de los operadores de escalera sobre el estado base.

Espero que esto haya servido de ayuda.

(TL;DR -- Si otro estado existiera, tendría una energía que no es de la forma de las dadas por los operadores de escalera. Sin embargo, actuando sobre este estado muchas veces con $a$ produciría una energía arbitrariamente baja, por lo que dicho estado no podría existir).

0 votos

Me gusta esta prueba porque tiene sentido intuitivo y responde a la pregunta. Sin embargo, esta prueba no es técnicamente innecesaria dado que sabemos que el espectro de H está formado por $\hbar \omega(n+1/2)$ para n>=0, y mostrar cómo se llega a cada estado propio utilizando el operador de creación que actúa sobre el estado básico? No quiero desmerecer la pregunta ni la respuesta, creo que ambas son muy buenas.

4 votos

Se trata de demostrar que las energías que has dado son las únicas posibles para este problema.

1 votos

@BobKnighton (1/3) Creo que uno de los puntos débiles de esta prueba es que aquí no has demostrado la unicidad general del estado base, salvo (de nuevo) el operador escalera tradicional ( $a$ ) y sus poderes ( $a^{m}$ ), cuya unicidad se pone en duda para empezar. Sí, $a|0>=0$ define una ecuación diferencial de primer orden, pero para los operadores $a$ . Otro punto es que, incluso después de demostrar la unicidad del estado base para tales operadores (no es difícil de hacer, ver la prueba aquí: physics.stackexchange.com/questions/380655/ ),

5voto

Jahan Claes Puntos 1026

Para demostrarlo, es necesario conocer el siguiente teorema: No hay estados ligados degenerados en 1D . La prueba se encuentra en el enlace. En el caso del potencial oscilante, cada estado propio es un estado ligado, por lo que no hay degeneración en el espectro.

Una vez que sabemos eso, el resto de la prueba es bastante sencilla. En lo que sigue, ignoraré la normalización y pondré $\hbar\omega=1$ para facilitar la notación. Tenemos un único estado básico, dado por $|0\rangle$ que satisface $a|0\rangle=0$ y $H|0\rangle=\frac{1}{2}|0\rangle$ . Sobre ese estado básico, definimos una torre de estados $|n\rangle=(a^\dagger)^n|0\rangle$ que satisfacen $H|n\rangle=(n+\frac{1}{2})|n\rangle$ . Afirmo que estos son todo los estados propios. Lo demostraremos por contradicción.

Digamos que hay algún otro estado propio $|\phi\rangle$ que no es uno de los $|n\rangle$ . Entonces $H|\phi\rangle=E_\phi|\phi\rangle$ . Utilizando la relación de conmutación de $a$ y $H$ se puede demostrar que $a^k|\phi\rangle$ es también un estado propio del Hamiltoniano con valor propio $E_\phi-k$ . Como sabemos que los valores propios no pueden ser negativos, eso significa que para algunos $k$ este proceso debe terminar. En otras palabras, para algún k, $a^k|\phi\rangle\neq 0$ pero $a^{k+1}|\phi\rangle=0$ . Pero si eso es cierto, $a^k|\phi\rangle$ necesariamente tiene energía $\frac{1}{2}$ ya que $Ha^{k}|\phi\rangle=(a^\dagger a+\frac{1}{2})a^{k}|\phi\rangle=\frac{1}{2}|\phi\rangle$ . Pero también sabíamos que este estado tenía energía $E_\phi-k$ . Así, $E_\phi-k=\frac{1}{2}$ o $E_\phi=k+\frac{1}{2}$ .

Pero eso es una contradicción. Eso significa que $|\phi\rangle$ y $|k\rangle$ tienen la misma energía. Por lo tanto, como no hay degeneración en 1D, deben ser el mismo estado. Esto contradice nuestra suposición de que $|\phi\rangle$ no estaba en nuestra lista inicial de estados.


EDIT: En respuesta a su confusión en los comentarios, tal vez un ejemplo concreto ayude. Digamos que hay un estado $|\frac{1}{2}\rangle$ . Entonces $a|\frac{1}{2}\rangle$ sería un estado $|-\frac{1}{2}\rangle$ y $a^2|\frac{1}{2}\rangle$ sería un estado $|-\frac{3}{2}\rangle$ etc. Esto no está permitido, porque sabemos que nuestro hamiltoniano sólo tiene vectores propios no negativos, pero, por ejemplo $H|-\frac{3}{2}\rangle=(-1)|-\frac{3}{2}\rangle$ .

La única manera de no obtener vectores propios arbitrariamente negativos es si en algún momento $a^k|\phi\rangle$ ya no es un estado válido de su sistema. ¿Qué es un estado no válido del sistema? El vector cero es el único vector del espacio de Hilbert que no es un estado permitido de tu sistema. Por lo tanto, en algún momento, usted necesita tener $a^{k+1}|\phi\rangle=0$ . Pero, como he argumentado anteriormente, al tener $a^{k+1}|\phi\rangle=0$ implica que $a^k|\phi\rangle$ necesariamente tiene energía $\frac{1}{2}$ . Esto significa que $|\phi\rangle$ tenía energía $k+\frac{1}{2}$ Así pues, es $|k\rangle$ . También significa que $a^k|\phi\rangle$ es en realidad sólo $|0\rangle$ ya que $|0\rangle$ es el único vector con energía $\frac{1}{2}$

En ninguna parte asume que $|\phi\rangle$ no era un estado como $|1.41\rangle$ . Pero nosotros probado que si tal estado existiera, podríamos utilizar operadores de escalera para generar toda una serie de estados con energías arbitrariamente negativas. Como sabemos que nuestro Hamiltoniano no tiene estados de energía negativa, eso implica que un estado propio como $|1.41\rangle$ no puede existir.

0 votos

Cuando escribes "En otras palabras, para algunos k, $a^k|\phi\rangle\neq 0$ pero $a^{k+1}|\phi\rangle=0$ .' , usted está asumiendo que $a^{k+1}|\phi\rangle$ es el estado básico. Puede que no sea así. Quiero decir que esta suposición no está justificada. Cuando se considera ese estado como el estado básico, entonces el problema en sí se resuelve. Puede ser que $a^{k+1}|\phi\rangle$ es un estado que se encuentra entre el estado básico y el primer estado excitado, como por ejemplo $|\frac12\rangle$ .

0 votos

Este es el punto principal de mi pregunta. Quiero saber si existen niveles de energía o estados energéticos como $|\pi\rangle$ , $|1.57\rangle$ , $|2.31\rangle$ entre estados como $|1\rangle$ , $|2\rangle$ , $|3\rangle$ etc. Y el nombre de estos estados no es sólo para una nomenclatura graciosa, sino que con esta denominación quiero denotar estados que realmente se encuentran entre $|n\rangle$ donde n es un número entero. O mejor dicho, los estados de energía que no se pueden encontrar utilizando el operador escalera.

0 votos

@SchrodingersCat ¡No estoy asumiendo eso en absoluto! $a^{k+1}|\phi\rangle$ no es un estado básico. No es un estado en absoluto. Es un $0$ . No $|0\rangle$ , pero en realidad $0$ como en el vector nulo.

4voto

Stefano Puntos 763
  1. Aquí supondremos que el oscilador armónico cuántico se da en un entorno algebraico (en lugar de geométrico). Digamos que sólo sabemos que
    $$\tag{1}\frac{\hat{H}}{\hbar\omega} ~:=~ \hat{N}+\nu{\bf 1}, \qquad\qquad \nu\in\mathbb{R},$$ $$\tag{2} \hat{N}~:=~\hat{a}^{\dagger}\hat{a}, $$ $$\tag{3} [\hat{a},\hat{a}^{\dagger}]~=~{\bf 1}, \qquad\qquad[{\bf 1}, \cdot]~=~0.$$ También suponemos que los estados físicos viven en un espacio de producto interno $(V,\langle \cdot,\cdot \rangle )$ .

  2. En mi respuesta de Phys.SE aquí con estos supuestos se demostró entonces que el espectro puntual del operador numérico $\hat{N}$ es precisamente todos los enteros no negativos $${\rm Spec}_p(\hat{N})~=~ \mathbb{N}_0 .\tag{4}$$

  3. En particular, es posible llegar a todos los niveles de energía (posiblemente degenerados) actuando sobre un estado de vacío con el operador de creación $\hat{a}^{\dagger}$ .

  4. Sin embargo, en el entorno algebraico (en lugar de geométrico) hay una advertencia: ¡el estado de vacío no tiene por qué ser único!

  5. Ejemplo: $$V~=~F_A\oplus F_B\tag{5}$$ podría ser una suma directa de dos espacios de Fock con estados de vacío $|0\rangle_A$ y $|0\rangle_B$ respectivamente. En este sistema un estado de vacío general es una combinación lineal de $|0\rangle_A$ y $|0\rangle_B$ . Nótese que entonces no sería posible transformar un estado de vacío fijo dado en un estado de energía arbitrario actuando únicamente con el operador de creación $\hat{a}^{\dagger}$ . También necesitamos a los otros estados de vacío.

1voto

valerio92 Puntos 483

Supongamos que $| m \rangle$ es un estado propio de $N=a^\dagger a$ es decir $N | m \rangle = m | m \rangle$ y que $m \notin \mathbb{N}$ .

Entonces, como

$$a | m\rangle = c | m-1\rangle$$

con $c \in \mathbb C$ para algunos $k \in \mathbb N$ tendremos

$$a^k | m\rangle = c' | m-k\rangle$$

con

$$m-k<0$$

A continuación demostraremos que se trata de un absurdum . En efecto, dado un valor propio $n$ de $N$ tenemos

$$n = \langle n | N | n \rangle = \langle n | a^\dagger a |n \rangle = \| a | n \rangle \|^2\geq 0$$

Concluimos entonces que nuestra hipótesis $m \notin \mathbb N$ debe estar equivocado, y que $m$ debe ser un número entero no negativo.

1voto

synergetic Puntos 1531

i-Ciencias.com

I-Ciencias es una comunidad de estudiantes y amantes de la ciencia en la que puedes resolver tus problemas y dudas.
Puedes consultar las preguntas de otros usuarios, hacer tus propias preguntas o resolver las de los demás.

Powered by:

X